1971 IMO Problems/Problem 3

Revision as of 12:40, 20 October 2013 by Elvis (talk | contribs) (Created page with "Prove that the set of integers of the form 2^k - 3(k = 2; 3; ...) contains an infinite subset in which every two members are relatively prime.")
(diff) ← Older revision | Latest revision (diff) | Newer revision → (diff)

Prove that the set of integers of the form 2^k - 3(k = 2; 3; ...) contains an infinite subset in which every two members are relatively prime.